latoyahi
Thanks Received: 0
Forum Guests
 
Posts: 2
Joined: March 25th, 2011
 
 
 

Q25 - A study of 86 patients,

by latoyahi Fri Mar 25, 2011 1:11 pm

I was able to reduce the answer choices down to A and C. However, I can not figure out why C is a choice than A. Please help...
 
Shiggins
Thanks Received: 12
Elle Woods
Elle Woods
 
Posts: 91
Joined: March 27th, 2011
 
This post thanked 4 times.
 
 

Re: Q25 - A study of 86 patients,

by Shiggins Sun Mar 27, 2011 5:02 pm

Hey, I also had trouble between A and C.
Choice A talks about the remaining four patients alive. They live more years, but this choice gives no indication about the other 82 members. 41 members of the non support groups could have lived longer.
Choice C takes into account the whole support group by taking the average into effect. If the average lived 2 more years then that is indicative of the support groups offering some advantage or help.
I hope this helps.
 
latoyahi
Thanks Received: 0
Forum Guests
 
Posts: 2
Joined: March 25th, 2011
 
 
 

Re: Q25 - A study of 86 patients...

by latoyahi Mon Mar 28, 2011 9:53 am

Thank you so much! I can see it now. Answer choice A is limited to the 4 patients that survived. Answer choice C applies to the group and therefore, "most weakens" the argument. :P
User avatar
 
bbirdwell
Thanks Received: 864
Atticus Finch
Atticus Finch
 
Posts: 803
Joined: April 16th, 2009
 
This post thanked 1 time.
 
 

Re: Q25 - A study of 86 patients...

by bbirdwell Tue Mar 29, 2011 11:37 am

Absolutely. Well done Shiggins!
I host free online workshop/Q&A sessions called Zen and the Art of LSAT. You can find upcoming dates here: http://www.manhattanlsat.com/zen-and-the-art.cfm
 
austindyoung
Thanks Received: 22
Elle Woods
Elle Woods
 
Posts: 75
Joined: July 05th, 2012
 
 
 

Re: Q25 - A study of 86 patients,

by austindyoung Wed Aug 29, 2012 11:42 pm

I think I had the same thought process..

The key, I think, is that the 10 years is an interval.

Within that 10 year frame, 41 from each group died. Maybe the ones in the support group died on the 10th year and the ones in the non-support group died after the 5th?

That's why (C) works- because if they all died at the same time (which one could easily think if reading too quickly after 10 years and BAM! they're dead)- then it wouldn't be possible for them to live 2 years longer on average. If we see it as a frame on a time line instead of a period on it- then it makes much more sense and (C) wins.

I guess it's obvious, but that's how I had to think of it.
 
timmydoeslsat
Thanks Received: 887
Atticus Finch
Atticus Finch
 
Posts: 1136
Joined: June 20th, 2011
 
This post thanked 1 time.
 
trophy
Most Thanked
trophy
First Responder
 

Re: Q25 - A study of 86 patients,

by timmydoeslsat Thu Aug 30, 2012 1:48 pm

You are absolutely right. The ten year mark gives us no indication as to the breakdown of the deaths per group during that time. Like you said, had the majority of the weekly support group died in the 10th year while the majority of the non-support group died much sooner than that, then we have support, forgive the pun, that the group stuff does help one live longer.
 
wj097
Thanks Received: 0
Atticus Finch
Atticus Finch
 
Posts: 123
Joined: September 10th, 2012
 
 
 

Re: Q25 - A study of 86 patients,

by wj097 Thu Nov 01, 2012 10:47 am

I completely follow the thought trail, but want to make sure how we get rid of (A)!

Is it because 1) it neither supports nor weakens (as we can't say anything about the 82 that died earlier) or 2) its a slight weakener (as we can say "oh at least we know that for 4 ppl, support group seemed to work"). Any help from LSAT geeks would be greatly appreciated it as its some delicate issue (I personally think).

I just feel like there can be Qs out there that has an answer like (A)...
 
monygg85
Thanks Received: 1
Jackie Chiles
Jackie Chiles
 
Posts: 29
Joined: December 04th, 2012
 
 
 

Re: Q25 - A study of 86 patients,

by monygg85 Tue May 14, 2013 1:23 pm

I'm curious about this question for the same reasons as the poster above. I see A as a "slight" weakener but C is better for sure as it incorporates everyone. Am I on the right track?
User avatar
 
tommywallach
Thanks Received: 468
Atticus Finch
Atticus Finch
 
Posts: 1041
Joined: August 11th, 2009
 
This post thanked 1 time.
 
 

Re: Q25 - A study of 86 patients,

by tommywallach Fri May 17, 2013 1:12 pm

Hey All,

Yes indeed. This is a rare question, because in the vast majority of cases, LSAT strengthen-weaken questions will have four totally wrong answers and one correct answer. In other words, a strengthen question will have four answer choices that either weaken or do nothing, and one that strengthens.

Very rarely, you'll get a question like this one, where two answer choices could be said to weaken.

However, I refuse to play into the whole "this is a tough call" game. Answer choice (A) and (C) are basically the same, except one of them relates to 4 people, while the other relates to 86 people. There's no comparison.

That being said--yes, both of them do weaken. : )

-t
Tommy Wallach
Manhattan LSAT Instructor
twallach@manhattanprep.com
Image
 
BensonC202
Thanks Received: 0
Vinny Gambini
Vinny Gambini
 
Posts: 19
Joined: April 08th, 2019
 
 
 

Re: Q25 - A study of 86 patients,

by BensonC202 Thu Jul 23, 2020 7:08 pm

Based on the argument, apparently it attempts to justify the causation based on the correlation.

If that's the case, would not answer D also weaken the argument by stating that there is the other alternative cause of " less faith " to have a group of patients attending support group to " not " live longer ?

If it is true most of whom attending weekly supporting group have less faith in dealing with disease T, then it must be true that there is other possible causes to make them " not " live longer than the other group. Perhaps group offering supports, however, due to faith lost, undesired to fight disease T no more, be negatively impacted by their own psychological state. Which perfectly weaken the argument, right ?

I can see both A and C could weaken the argument in a sense that they both raise the other correlations that counter the original one. However, if C is correct answer, would not we must to assume that there is no any other exterior force to have patients from the weekly supporting group to live longer ? Also, what if within 43 patients of attending the weekly support group from the first date whom participated in the study " are " 2 years older than the other group of patients without attending the group ? Based on C, it does not explicitly indicate the term of " 2 years longer of life lived " happened within that 10 years segment. Which is to say, if answer C clearly stated that within 10 years, the average life span of whom attending supporting group is 2 years more than the other group, and both the negative and positive impact derived from standard deviation of the age of both group be discerned. Then C might be regarded as " potentially correct answer "

In terms of answer A, it attempts to equate the characteristic ( live longer ) of few samples to it of the whole group, and the attempt serves the best reason to be eliminated in the first place.

Perhaps I am absolutely wrong within some of my reasoning process; however, please help me out to improve.